Como podemos limitar a probabilidade de uma variável aleatória ser máxima?


21

Suponha que tenhamos variáveis ​​aleatórias independentes , , com meios finitos e variâncias , , . Estou procurando limites livres de distribuição com a probabilidade de que qualquer seja maior que todos os outros , .X 1X n μ 1μ N σ 2 1σ 2 N X iX N X j j iNX1Xnμ1μNσ12σN2XiXNXjji

Em outras palavras, se por simplicidade assumimos que as distribuições de Xi são contínuas (de modo que P(Xi=Xj)=0 ), estou procurando limites em:

P(Xi=maxjXj).
Se N=2 , podemos usar a desigualdade de Chebyshev para obter:
P(X1=maxjXj)=P(X1>X2)σ12+σ22σ12+σ22+(μ1μ2)2.
Gostaria de encontrar alguns limites simples (não necessariamente restritos) para o N geral N, mas não consegui encontrar resultados esteticamente agradáveis ​​para o N geral N.

Observe que as variáveis ​​não são consideradas iid. Quaisquer sugestões ou referências a trabalhos relacionados são bem-vindas.


Atualização: lembre-se de que, por suposição, μjμi . Podemos então usar o limite acima para chegar a:

P(Xi=maxjXj)minj>iσi2+σj2σi2+σj2+(μjμi)2σi2+σN2σi2+σN2+(μNμi)2.
Isso implica:
(μNμi)P(Xi=maxjXj)(μNμi)σi2+σN2σi2+σN2+(μNμi)212σi2+σN2.
Isso, por sua vez, implica:
i=1NμiP(Xi=maxjXj)μNN2i=1N1(σi2+σN2).
Agora estou pensando se esta ligado pode ser melhorado para algo que não depende linearmente em N . Por exemplo, o seguinte é válido:
i=1NμiP(Xi=maxjXj)μNi=1Nσi2?
E se não, o que poderia ser um contra-exemplo?

3
Este limite pode ser mais apertado, se você usar o índice que lhe dá o menor limite superior, em vez de . Observe que esse valor depende da média e da variação. jN

5
@ MichaelChernick: Não acredito que isso esteja correto. Suponha, por exemplo, que tenhamos três distribuições uniformes em . Então, se não me engano, , enquanto . Não sei se você pretendia escrever , mas o mesmo exemplo mostra que ainda não é válido. [0,1]P(X1<maxjXj)=2/3P(X1<X2)=P(X1<X3)=1/2P(Xi>maxjXj)
MLS

2
@ Michael: Isso ainda não é verdade, infelizmente. Os eventos para fixo não são independentes. Aj={Xi>Xj} i
cardeal

2
@ cardinal: Entre outras coisas, está relacionado a bandidos com várias armas. Se você escolher um braço com base nas recompensas anteriores, qual a probabilidade de escolher o melhor braço (que seria na notação acima) e podemos limitar a perda esperada ao escolher um sub braço ideal? P(XN=maxjXj)
MLS

2
Crossposted to MathOverflow: mathoverflow.net/questions/99313
cardinal

Respostas:


1

Você pode usar a desigualdade de Chebyshev multivariada.

Caso de duas variáveis

Para uma situação única, x , chego à mesma situação que o comentário de Jochen em 4 de novembro de 2016X1X2

1) Se entãoμ1<μ2P(X1>X2)(σ12+σ22)/(μ1μ2)2

(e também me pergunto sobre sua derivação)

Derivação da equação 1

  • usando a nova variávelX1X2
  • transformando-o de modo que tenha a média em zero
  • tomando o valor absoluto
  • aplicando a desigualdade de Chebyshev

P(X1>X2)=P(X1X2>0)=P(X1X2(μ1μ2)>(μ1μ2))P(|X1X2(μ1μ2)|>μ2μ1)σ(X1X2(μ1μ2))2(μ2μ1)2=σX12+σX22(μ2μ1)2

Caso Multivariado

A desigualdade na equação (1) pode ser alterada para um caso multivariado, aplicando-a a múltiplas variáveis ​​transformadas para cada i < n (observe que elas estão correlacionadas).(XnXi)i<n

Uma solução para esse problema (multivariada e correlacionada) foi descrita por I. Olkin e JW Pratt. 'Uma desigualdade multivariada de Tchebycheff' nos Anais de estatística matemática, volume 29 páginas 226-234 http://projecteuclid.org/euclid.aoms/1177706720

Teorema da nota 2.3

P(|yi|kiσi for some i)=P(|xi|1 for some i)(u+(ptu)(p1))2p2

em que o número de variáveis, t = Σ k - 2 i , e u = Σ p i j / ( k i k j ) .pt=ki2u=ρij/(kikj)

O teorema 3.6 fornece um limite mais rígido, mas é menos fácil de calcular.

Editar

Um limite mais nítido pode ser encontrado usando a desigualdade de Cantelli multivariada . Essa desigualdade é o tipo que você usou anteriormente e forneceu o limite que é mais nítido que ( σ 2 1 + σ 2 2 ) / ( μ 1 - μ 2(σ12+σ22)/(σ12+σ22+(μ1μ2)2) .(σ12+σ22)/(μ1μ2)2

Não tomei tempo para estudar o artigo inteiro, mas, de qualquer maneira, você pode encontrar uma solução aqui:

AW Marshall e I. Olkin 'Uma desigualdade unilateral do tipo Chebyshev' em Annals of Mathematics Statistics volume 31 pp. 488-491 https://projecteuclid.org/euclid.aoms/1177705913

(nota posterior: essa desigualdade é para correlações iguais e não ajuda suficiente. Mas, de qualquer maneira, seu problema, para encontrar o limite mais nítido, é igual à desigualdade Cantelli, mais geral e multivariada. Eu ficaria surpreso se a solução não existir)


Could you provide a clear statement of the multivariate Chebyshev Inequality?
whuber

1
I have edited the solution providing the entire theorem.
Sextus Empiricus

-1

I have found a theorem that might help you and will try to adjust it for your needs. Assume you have:

exp(tE(max1inXi))

Then by Jensen's inequality (since exp(.) is a convex function), we get:

exp(tE(max1inXi))E(exp(tmax1inXi))=E(max1in exp(tXi))i=1nE(exp(tXi)

Now for exp(tXi you have to plug in whatever the moment generating function of your random variable Xi is (since it is just the definition of the mgf). Then, after doing so (and potentially simplifying your term), you take this term and take the log and divide by it by t so that you get a statement about the term E(max1inXi). Then you can choose t with some arbitrary value (best so that the term is small so that the bound is tight).

Then, you have a statement about the expected value of the maximum over n rvs. To get now the a statement about the probabilty that the maximum of those rv's deviates from this expected value, you can just use Markov's inequality (assuming that your rv is non-negative) or another, more specific rv, applying to your particular rv.

Ao utilizar nosso site, você reconhece que leu e compreendeu nossa Política de Cookies e nossa Política de Privacidade.
Licensed under cc by-sa 3.0 with attribution required.